LSAT and Law School Admissions Forum

Get expert LSAT preparation and law school admissions advice from PowerScore Test Preparation.

 Administrator
PowerScore Staff
  • PowerScore Staff
  • Posts: 8917
  • Joined: Feb 02, 2011
|
#82105
Complete Question Explanation

The correct answer choice is (E).

Answer choice (A):

Answer choice (B):

Answer choice (C):

Answer choice (D):

Answer choice (E): This is the correct answer choice.


This explanation is still in progress. Please post any questions below!
 Nicholas Noyes
  • Posts: 38
  • Joined: Feb 07, 2020
|
#75158
Hi,

Could you please explain why Fredericks, Guerrero, Horowitz, and Lee could all be the architects of X? I am having trouble figuring this question out.

-Nicholas
 Paul Marsh
PowerScore Staff
  • PowerScore Staff
  • Posts: 290
  • Joined: Oct 15, 2019
|
#75182
Hi Nicholas! Let's walk through this one.

As posted in the Setup thread, this is the main diagram of the game:

https://forum.powerscore.com/lsat/viewtopic.php?t=17363

This question is asking us for the complete list of architects (out of F, G, H, L) who could be assigned to X. For these types of questions, remember that diagrams from Local questions and the initial List question can be very helpful at narrowing down the answer choices. Unfortunately, we don't have many Local questions for this game. However, our answer to question 7 at least informs us that F can be assigned to X. What about the other 3 architects? Since we have no rules about X and have made no inferences about it in our main diagram, there is no reason to think that any of the other 3 architects can't be assigned to X. As far as I can tell, there is no faster way to truly solve this problem other than testing out each of the other 3 architects. If I was incredibly pressed for time, I would probably trust in my (lack of) inferences about who can/can't be assigned to X and the fact that X is a floater and just put answer Choice (E). But since we've got the time, let's test them out.

If G is assigned to X, then we know from Rule 2 that F is assigned to W. That means, from rule 1, that L is assigned to Z. That leaves H with Y. As always, H goes last, L will either go 1st or 4th, with F/G going 2/3 or 3/4. So that possibility would look like:

/(Z) W X /(Z) Y
/(L) F G /(L) H

That possibility makes sense and doesn't violate any of our rules.

What about if H is assigned to X? Well, we then know from the contrapositive of our third rule that F is assigned to W. From our first rule, we then know that L is assigned to Z. That leaves G to be assigned to Y. As always, H goes last, L will either go 1st or 4th, with F/G going 2/3 or 3/4. So that possibility would look like:

/(Z) W Y /(Z) X
/(L) F G /(L) H

And that violates none of our rules.

Last possibility - L is assigned to X. That means, from rule 1, that F is assigned to Z. Since F is not assigned to W, rule 3 and the contrapositive of rule 2 tell us that H is assigned to Y and G is assigned to W. As always, H goes last, L will either go 1st or 4th, with F/G going 2/3 or 3/4. So that possibility would look like:

/(X) Z W /(X) Y
/(L) F G /(L) H

That possibility checks out, and violates none of our rules.

So we can assign any of the architects to X and it wouldn't violate any of our rules. Tha

Hope that helps!
User avatar
 hotdiogenes
  • Posts: 1
  • Joined: Sep 26, 2021
|
#90790
I don't understand how choice E does not violate the contrapositive of rule 3 "h is assigned Y if f is not assigned W". If we reverse that rule to take the contrapositive then we have "if f is assigned W, h is not assigned to Y".

If we have gX, then the rest of the spread is lZ, fW, hY

But how can we have hY and fW at the same time? How am I supposed to know when rules require this kind of mutual exclusive???

Sincerely,
Confused
User avatar
 atierney
PowerScore Staff
  • PowerScore Staff
  • Posts: 215
  • Joined: Jul 06, 2021
|
#90879
Hi Confused,

The contrapositive would be "if h is not assigned y, then f is assigned w."

In other words we can assign f to w, and then assign h to x, l to z, and g to y.

With respect to g assigned to x, you would need to assign f to w, l to z, and then you can assign h to y in this case, because the reversal of the above contrapositive does not follow from the rules, "if f is assigned to w, then h is not assigned to y," and indeed would be a mistaken negation of the original rule.

Get the most out of your LSAT Prep Plus subscription.

Analyze and track your performance with our Testing and Analytics Package.